User avatar
 
bbirdwell
Thanks Received: 864
Atticus Finch
Atticus Finch
 
Posts: 803
Joined: April 16th, 2009
 
This post thanked 1 time.
 
 

Q10 - A fundamental illusion in robotics

by bbirdwell Thu Mar 11, 2010 2:42 pm

This question is hard because the correct answer is not what you would expect.

Now, since we are looking for a logical flaw, we must find the conclusion and the premise(s).

The conclusion is:
Robots will not eliminate demeaning work -- only substitute one type of demeaning work for another.

The evidence on which this conclusion is based is:
Engineers are only designing robots that can be properly maintained with the least expensive, least skilled human labor possible.

Now, when I first read this question, I thought the most obvious flaw was the assumption that "least expensive...least skilled" was equivalent to "demeaning."

(A) out of scope. way out.
(B) tempting. Actually, though, this is untrue. The author does not assume that robots create demeaning work - the author provides an example of how that work is needed to maintain robots. Therefore, it's stated, not assumed.
(C) doesn't matter how the engineers feel about their jobs.
(D) way out of scope.
(E) Consider this answer choice. IF it is true that the amount of demeaning work eliminated by the robots is GREATER than the amount they create, the conclusion (that robots will not eliminate demeaning work) is clearly false -- the sum total of demeaning work in the world will have decreased because of the robots.

I can see where one might get tangled up with "eliminate," but ultimately the word "eliminate" does not mean "eliminate ALL." When you hear that GE is "eliminating jobs," it doesn't mean they fired EVERYONE.
I host free online workshop/Q&A sessions called Zen and the Art of LSAT. You can find upcoming dates here: http://www.manhattanlsat.com/zen-and-the-art.cfm
 
shaynfernandez
Thanks Received: 5
Elle Woods
Elle Woods
 
Posts: 91
Joined: July 14th, 2011
 
 
 

Re: Q10 - A fundamental illusion in robotics

by shaynfernandez Wed Jun 13, 2012 3:03 pm

Well, I came on here to try to voice my frustration with this question and resolve some misunderstandings. BUT of course I come on here and there is already a crystal clear explanation which addresses all my problems. So thank you!

I do see why E is correct because it's an answer that is commonly correct in these ID flaw questions with causes as well as addresses the gap in amount. But with choice B I had a hard time eliminating it because it did appear to be an assumption, though like you said it was actually a premise. I come across this choice a lot in flaw questions and was wondering if you could provide me with an example of what "assuming what it sets out to prove" would look like. It seems like this is kind of a contradiction in and of itself. If what the author assumes also shows up in the conclusion it's not really an assumption anymore. Right?

Also what you said about "eliminate" also caught me during this. When I think of eliminate I think of completely remove, which is its definition. That made answer choice E very unsettling.
 
griffin.811
Thanks Received: 43
Atticus Finch
Atticus Finch
 
Posts: 127
Joined: September 09th, 2012
 
 
 

Re: Q10 - A fundamental illusion in robotics

by griffin.811 Wed Jan 09, 2013 1:07 am

Great explanation, but not a fan of this question. The wording is very misleading. I know that is the goal of the test writers, but, this use of the word "eliminate" is a completely different type of trickery :twisted: .

Glad I stumbled across it before the test.
 
dmt137
Thanks Received: 0
Forum Guests
 
Posts: 5
Joined: November 02nd, 2013
 
 
 

Re: Q10 - A fundamental illusion in robotics

by dmt137 Sat Nov 02, 2013 7:04 pm

I'm confused by the use of the word "substitute" in the stimulus. Even if the newly created demeaning work is significantly less than the work it is eliminating, like answer choice E says, it is still a "substitute" regardless of the amount. This made me think that choice didn't weaken the argument at all...
User avatar
 
WaltGrace1983
Thanks Received: 207
Atticus Finch
Atticus Finch
 
Posts: 837
Joined: March 30th, 2013
 
 
trophy
Most Thanked
trophy
Most Thankful
trophy
First Responder
 

Re: Q10 - A fundamental illusion in robotics

by WaltGrace1983 Wed Nov 12, 2014 10:01 pm

dmt137 Wrote:I'm confused by the use of the word "substitute" in the stimulus. Even if the newly created demeaning work is significantly less than the work it is eliminating, like answer choice E says, it is still a "substitute" regardless of the amount. This made me think that choice didn't weaken the argument at all...


I agree with your frustration. However, I think "substitute" here is supposed to be taken as an equal substitute, no more and no less. This is a pretty old test though so maybe it is a bit rusty in comparison to the modern ones.
 
Dtodaizzle
Thanks Received: 0
Vinny Gambini
Vinny Gambini
 
Posts: 24
Joined: February 08th, 2015
 
 
 

Re: Q10 - A fundamental illusion in robotics

by Dtodaizzle Tue Jul 21, 2015 11:08 pm

Since the most logical error is the term shift between "The least expensive, least skilled human labor possible" and "demeaning", I was wondering if the statement that I created below would address this flaw.


The majority of the population and robotic experts consider maintaining robots to be a highly complex task, and expenditures paid to servicing the robot could be prohibitively expensive.


Would a statement like the one above address the term shift and be a viable answer?
User avatar
 
maryadkins
Thanks Received: 640
Atticus Finch
Atticus Finch
 
Posts: 1261
Joined: March 23rd, 2011
 
 
 

Re: Q10 - A fundamental illusion in robotics

by maryadkins Fri Jul 31, 2015 10:35 am

Two things:

1) It doesn't really matter what people think. That's subjective. What matters is linking OBJECTIVELY "demeaning" to "least expensive" and "least skilled."

2) More importantly, we're already told that the robots being designed require low cost, low skilled labor to maintain. We can't challenge that. It's a premise.

So for both of these reasons it wouldn't be a viable answer.